you are evenly dividing up 57 apples among 5 friends

Answers

Answer 1

Answer:

11.4 have a great day :)

Step-by-step explanation:

Answer 2

Answer:

This cannot be true.

Step-by-step explanation:

You cannot divide 57 evenly among 5 because 57 is not a multiple of five therefore 57 can't be divided by 5.


Related Questions

PLEASE HELP SOON!
Simplify the inequalities and match them with the graphs that represent them.
(look at the screen shot)

Answers

Answers:

Graph A matches with 7x < 42Graph B matches with x+1 < 8Graph C matches with 3x > 15Graph D matches with 2x > 12

===================================================

Explanation:

For each inequality given, we need to isolate x. We'll solve for x in pretty much the same way we solve an equation. So if we wanted to solve 2x = 12, we would divide both sides by 2 to get x = 6.

Similarly, solving 2x > 12 would lead to x > 6. The inequality sign does not flip. It only flips if you divide both sides by a negative number.

The graph of x > 6 has an open hole at 6 and shading to the right. So that matches with graph D.

---------------------

Solving 7x < 42 leads to x < 6 when we divide both sides by 7. The graph has the open hole at the same endpoint of 6, but now the shading is to the left. This shows all values smaller than 6. Therefore 7x < 42 matches with graph A.

---------------------

We have 3x > 15 match with graph C because that inequality is the same as x > 5, after we divide both sides by 3. We have an open hole at 5 and shading to the right.

---------------------

Lastly, the inequality x+1 < 8 solves to x < 7. We'll have an open hole at 7 and shading to the left. This describes graph B.

Any time you see an open hole, it means "do not include this value as part of the solution set". For instance, with x < 7, we have x = 7 as the open hole. This is because 7 < 7 is a false statement. A number cannot be smaller than itself.

Please help!!!!!!!!!!!!!!!!!!!!

Answers

Answer:

b is 100

Step-by-step explanation:

Answer:

Average is 100

Median is 87.5

Step-by-step explanation:

For average you add up all numbers and divide by how many there are, in this case all numbers added together equals 600, then you divide by 6.

For median, you put the valuse from least to greatest and find the number in the middle, for these numbers, there are 2, 75 and 100, so you add those together and divide them by 2.

Hi can you please help I’ve been stuck on this question

Answers

9514 1404 393

Answer:

  20 meters

Step-by-step explanation:

Using the formula for the area of a rectangle, fill in the given information and solve for the unknown.

  A = LW . . . . . . . . area is length × width

  80 m² = L·(4 m)

  (80 m²)/(4 m) = L = 20 m . . . . . . . divide by 4m (the coefficient of L)

The tarp is 20 meters long.

The height of a ball t seconds after it's thrown into the air
from the top of a building can be modeled by h(t) = -16t? +
48t + 64, where h(t) is height in feet. How high does the
ball rise before starting to drop downward?
48 ft
O 100 ft
O 116 ft
O 64 ft

Answers

Answer:

The first one is: H=-6 and t=0

The second one is:16x(3t+4)

And i think the answer is:116

Step-by-step explanation:

Correct me if i'm wrong

What is the midpoint between -9 and 15 on the number line?

12
2
3
6

Answers

3 I could be wrong but hopefully it is right

In a mathematics class, half of the students scored 55 on an achievement test. With the exception of a few students who scored 72, the remaining students scored 60. Which of the following statements is true about the distribution of scores?
A. The mean and the median are the same.
B. The mean is greater than the median.
C. The mean is less than the mode.
D. The mean is less than the median.

Answers

Answer: B



Step-by-step explanation: Trust me

Mean (Average) 62.333333333333

Median 60

oWhat can I say execpt your welcome

The mean is greater than the median.

Option (B) is correct.

To find the statements is true.

What is statistics?

A branch of mathematics dealing with the collection, analysis, interpretation, and presentation of masses of numerical data.

Half the students scored 55.

The next highest score is 60. So the median is (55+60) / 2 = 57.5.

A few students scored 72, so the mean is slightly greater than the mean of 55 and 60.

Therefore, the mean is greater than the median.

Learn more about statistics here:

https://brainly.com/question/22613249

#SPJ2

PLEASE HELP THIS IS AN EMERGENCY!! Given f(x) = 3x2 - 6x + k, and the
remainder when f (x) is divided by X – 1
is 1, then what is the value of k?

Answers

Answer:

The values of

k are

(-4,2)

Step-by-step explanation:

sorry if wrong

Lamar needs to buy some pencils. Brand A has a pack of 18 pencils for 2.36 . Brand B has a pack of pencils 24 for 4.09 Find the unit price for each brand. Then state which brand is the better buy based on the unit price. Round your answers to the nearest cent.

Answers

Answer:

The unit price of brant A and brand B is 0.1311 and 0.1704 and brand A would be better

Step-by-step explanation:

The computation of the unit price for each brand is shown below:

For brand A, the unit price is

= 2.36 ÷ 18 pencils

= 0.1311

For Brand b, the unit price is

= 4.09 ÷ 24

= 0.1704

So here as we can see that brand A would be better as it has less cost as compared with brand B

Can you guys help me with this please it suppose to use the synthetic substitution or the remainder theorem.

Answers

9514 1404 393

Answer:

  (b)  -36

Step-by-step explanation:

Synthetic division is often performed using a table of the kind shown in the attachment. The coefficients of f(x) are placed across the top, and the x-value to be evaluated is placed at the left. The rule for finding table entries is shown. (The first (left-most) number at the bottom is the leading coefficient of the polynomial.)

This table shows the remainder, which is f(-2), is -36.

__

The equation itself can be rewritten in "Horner form" to facilitate evaluation.

  f(x) = (((-x +2)x -1)x +4)x +8

In this form, substituting for x and doing the evaluation is easier for mental arithmetic. You will notice that the contents of parentheses at each level match the bottom line of the synthetic division table.

  f(-2) = (((-(-2) +2)(-2) -1)(-2) +4)(-2) +8

  = (((4)(-2) -1)(-2) +4)(-2) +8

  = ((-9)(-2) +4)(-2) +8

  = (22)(-2) +8

  = -36

  f(-2) = -36 . . . . remainder from synthetic substitution

Can someone help me with this ASAP please I’m being timed

Answers

Answer:

false

Step-by-step explanation:

This is because the ones that most likely have a computer are going to play and the ones that dont arent going to play.

It is False you are welcome

How are you really? 100 points, Be honest.

Answers

Answer:

Why are you wasting your points? LOL XD

Step-by-step explanation:

Answer:

Good

Step-by-step explanation:

The figure is not drawn to scale. PQ and MN are straight lines.
Find ZZ
helpppp

Answers

180-135=45

45+z+87=180
132+z=180
z=48

2359 million in standard form

Answers

Answer:

2,359,000,000

Step-by-step explanation:

Answer:

2,359,000,000 would be the standard form

Step-by-step explanation:

10x + 5y = -5 and y= -2x + 6

slope 1st line
slope 2nd line
Type of Lines​

Answers

Answer:

-2 , -2, parallel

Step-by-step explanation:

same slope, different y intercept, parallel

What is the next fraction in this sequence? Simplify your answer. 4 5 , 2 3 , 8 15 , 2 5 , ...

Answers

Answer:

4/15

Step-by-step explanation:

To simplify this question, to make it easier to be solved, we try and make sure that all the fractions have the same denominator. This is done by saying

4/5

2/3

8/15

2/5

From the aforelisted numbers, the highest denominator is 15, and thus, we try to make all the denominators 15

4/5 * 3/3 = 12/15

2/3 * 5/5 = 10/15

8/15 * 1/1 = 8/15

2/5 * 3/3 = 6/15

I multiplied it that way, so that it's decimal value doesn't change, even as it gives us the required fraction.

From the new crop of fractions, we have

12/15

10/15

8/15

6/15

If there's anything, we can tell that the sequence is reducing in a -2/15 rate, and thus, our next sequence has to be 4/15

Write the equation for a line in Slope-Intercept Form (y = mx +b) given the two points below (2,10) (3,-5)

Answers

Given:

The two end points of a line are (2,10) and (3,-5).

To find:

The equation of the line in the slope intercept form.

Solution:

The slope intercept form of a line is

[tex]y=mx+b[/tex]

Where, m is slope and b is the y-intercept.

The two end points of a line are (2,10) and (3,-5). So, the equation of the line is

[tex]y-y_1=\dfrac{y_2-y_1}{x_2-x_1}(x-x_1)[/tex]

[tex]y-10=\dfrac{-5-10}{3-2}(x-2)[/tex]

[tex]y-10=\dfrac{-15}{1}(x-2)[/tex]

[tex]y-10=-15(x-2)[/tex]

On further simplification, we get

[tex]y-10=-15x+30[/tex]

[tex]y=-15x+30+10[/tex]

[tex]y=-15x+40[/tex]

Therefore, the slope intercept form of the given line is [tex]y=-15x+40[/tex].

HELPPPPP FAST PLEASEEEEEEEEEEEE

Answers

Answer:

45?

Step-by-step explanation:

Ethan once heard that wearing colorful socks helped release your positive energy toward others. Being the scientist that he is, he decided to put the claim to the test by running a well-designed study. He had 18 people wearing colorful socks at an auditorium where the rest of the people were wearing white socks. After carefully designing, collecting, and analyzing his data, he found no statistically significant difference between his sample of 18 participants and the rest of the population he studied regarding positive energy. What should Ethan conclude regarding the null hypothesis

Answers

Answer:

The answer is "Choice A".

Step-by-step explanation:

Please find the complete question in the attachment file.

Throughout this question by acceptance of its zero hypotheses becomes equal to "the failure to accept the zero hypotheses." Its best answer is to refuse the null hypothesis or conclude how he had no effects on vibrant socks mostly on basis of this study, that's why the above given choice is correct.

length and breadth of a cuboid are 13 cm and 8 cm respectively if the volume of a cube it is 40 cm find the height of the cubiod​

Answers

Answer:

Height of cuboid = 0.384 cm (Approx.)

Step-by-step explanation:

Given:

Length of cuboid = 13 cm

Width of cuboid = 8 cm

Volume of cuboid = 40 cm³

Find:

Height of cuboid

Computation:

Volume of cuboid = length x width x height

13 x 8 x Height of cuboid = 40

Height of cuboid = 0.384 cm (Approx.)

Resolver para u.
10u= bu + 20
Simplificar la respuesta tanto como sea posible.
U =
.
.
x 5 ?

Answers

10u = bu + 20
10u-bu=20
u(10-b)=20
u=20/(10-b)

PLSSS HELPPPP I WILLL GIVE YOU BRAINLIEST!!!!!! PLSSS HELPPPP I WILLL GIVE YOU BRAINLIEST!!!!!! PLSSS HELPPPP I WILLL GIVE YOU BRAINLIEST!!!!!! PLSSS HELPPPP I WILLL GIVE YOU BRAINLIEST!!!!!!PLSSS HELPPPP I WILLL GIVE YOU BRAINLIEST!!!!!! PLSSS HELPPPP I WILLL GIVE YOU BRAINLIEST!!!!!! PLSSS HELPPPP I WILLL GIVE YOU BRAINLIEST!!!!!!

Answers

Answer:

Its 268.08 hope this helps

Step-by-step explanation:

The correct answer and volume to this is 268.08 CM

Im 13 and 2 months years old and im 5'2 my brothers 5'11 and my mothers 4'11 how tall will i be
ANYONE WHO COULD PREDICT MY HEIGHT WILL GET BRAINLIEST!!

Answers

Answer:

5'2

Step-by-step explanation:

Find the values of x and y so that GHJK is a dart. Be sure to show all work .

Answers

Answer:

Step-by-step explanation:

In dart, adjacent sides are equal

JK = GK

3x - 9 = x + 13            {Add 9 to both sides}

3x      = x + 13 + 9

    3x = x + 22            {Subtract 'x' from both sides}

3x -x = 22

 2x  = 22           {Divide both sides by 2}

   x = 22/2

x = 11

JH = GH

9y = 5y + 8  {Subtract 5y from both sides}

9y - 5y = 8

    4y = 8        {Divide both sides by 4}  

  y   = 8/4

y = 2

HELP ONLY ANSWER IF YOU ARE 100% RIGHT. HAVE BEEN WORKING ON THIS FOR ALMOST 11 HOURS. SHOW WORK PLEASE!!!!!! NO LINKS!!!!

Answers

Answer:

180 ft2

Step-by-step explanation:

First solve for the area of the trapzeoid using formula:

A= a+b/2xh

10+18x6/2

A= 84 ft2

Then solve for the rectangles B and C:

12x4= 48

48x2 (because theres two rectangles) which equals 96

Combine the two to get the total area:

96+84= 180

PLEASE HELP!!!!! 50 POINTS!!!! REAL ANSWERS ONLY!!!!!! PLEASE HELP!!!!!! I NEED TO GET THIS DONE

finding angle measures with trig. Find the missing angle measures. Round to the nearest whole number​​

Answers

Answer:

#1

30° as opposite side is half of the hypotenuse

#2

7/16 = tan xx = arctan (7/16) = 24°

#3

31/50 = tan xx = arctan (31/50) = 32°

#4

5/7 = cos xx = arccos (5/7) = 44°

#5

4/14 = sin xx = arcsin (4/14) = 17°

#6

6/7 = cos xx = arccos (6/7) = 31°

#7

29/37 = tan xx = arctan (29/37) = 38°

#8

43/60 = sin xx = arcsin (43/60) = 46°

#9

32/44 = cos xx = arccos (32/44) = 43°

Answer:

let angle be x

Step-by-step explanation:

relationship between perpendicular and hypotenuse is given by sin angle

sin x =p/h

sin x=13/26

x=sin-¹(13/26)=30°

2.relationship between perpendicular and base is given by tan angle

tan x =p/b

tan x =7/16

x=tan -¹(7/16)=23.63°

3.relationship between between perpendicular and base is given by tan angle

tan x =p/b

tanx=31/50

x=tan-¹(31/50)=31.8°

4.relationship between base and hypotenuse is given by cos angle

cos x =b/h

cos x=5/7

x=cos-1(5/7)=44.4°

5.relationship between perpendicular and hypotenuse is given by sin angle

sin x =p/h

sin x=4/14

x=sin-¹(4/14)=16.6

6..relationship between base and hypotenuse is given by cos angle

cos x=6/7

x=cos-¹(6/7)=31.0027

7.relationship between perpendicular and base is given by tan angle

.tanx=29/37

x=tan-¹(29/37)=38.09

8.relationship between perpendicular and hypotenuse is given by sin angle

.sin x=43/60

x=sin-¹(43/60)=45.78

9..relationship between base and hypotenuse is given by cos angle

cos x=32/44

x=cos-¹(32/44)=43.34

The hypotenuse of a 30 -60 -90 triangle is 13cm. What is the length of the shorter leg?

Answers

Answer:

(13/2) cm

Step-by-step explanation:

The hypotenuse here is of length 13 cm.  The shorter leg lies opposite to the 30° angle.  Recall that the sine function is defined as

              opposite side

sin Ф = -----------------------

                 hypotenuse

which, in this case, works out to:

                opposite side

sin 30° = -----------------------

                       13 cm

and so the length of the opposite side (which is also the "shorter leg") is

shorter leg length = (13 cm)(sin 30°) = (13 cm)(1/2) = (13/2) cm

if y=3 sin (-4x), find dy/dx​

Answers

[tex]\mathfrak{\huge{\orange{\underline{\underline{AnSwEr:-}}}}}[/tex]

Actually Welcome to the Concept of the derivatives.

Hence, y = 3 sin(-4x), we use the chain rule here.

dy/dx =3 ( cos(-4x).-4)

====> dy/dx = -12 cos(-4x)

For ABC, let c = 600 ft, m angle B = 50° and then m angle A =30° Then a= ft. Round to nearest whole number.

Answers

Answer: 305 ft

Step-by-step explanation:

Given

[tex]\angle A=30^{\circ}[/tex]

[tex]\angle B=50^{\circ}[/tex]

[tex]c=600\ ft[/tex]

From the angle sum property of the triangle, we can write

[tex]\Rightarrow \angle A+\angle B+\angle C=180^{\circ}\\\Rightarrow \angle C=180^{\circ}-30^{\circ}-50^{\circ}\\\Rightarrow \angle C=100^{\circ}[/tex]

Now using Sine law i.e.

[tex]\dfrac{a}{\sin A}=\dfrac{b}{\sin B}=\dfrac{c}{\sin C}[/tex]

[tex]\Rightarrow \dfrac{a}{\sin 30^{\circ}}=\dfrac{600}{\sin 100^{\circ}}\\\\\Rightarrow a=600\times \dfrac{\sin 30^{\circ}}{\sin 100^{\circ}}=600\times 0.5077\\\\\Rightarrow a=304.62\ ft\approx 305\ ft[/tex]

don't know it is complicated

please answer and explain if you can ​

Answers

Answer:

no s e

s

Step-by-step explanation:

Answer:

its b and i don't know if you can see the screenshot

Step-by-step explanation:

If two cylinders have the same base, and one cylinder has a height of 4 inches and a volume of 120 cubic inches, and the second cyclinder has 7 inches what's the volume of the second cylinder

Answers

Answer:

210 cubic inches

Step-by-step explanation:

The volume V of  a cylinder is given as

V = πr^2h where r is the base radius and h is the height

If two cylinders have the same base, then their radii are the same

Given that one cylinder has a height of 4 inches and a volume of 120 cubic inches then

120 = πr^2 * 4

Divide both sides by 4

30 = πr^2

Given that the second cylinder has a height of 7 inches then the volume

V = πr^2 * 7

Recall that 30 = πr^2

V = 30* 7

= 210 cubic inches

Other Questions
6+(1/31/3)-37 how do you get it??? Please help! would appreciate it Find the indicated angle measure.Find mZFEHFG3540HE PLZ HELP!!! Select the correct sentence.Her daughter-in-laws house had a make yourself at home look.Her daughter-in-law's house had a make-yourself-at-home look. que sinifica afilado Write the equation of the line in slope-intercept form using y=mx+b 6.Why are estuaries home to diverse marine life not found elsewhere?is hrimming with Find the missing information for both parts below.(a) In the figure below, m VW = 52 and m XY = 42. Find m ZXZY.Vm ZXZY =Z please help ill give brainliest if you help thanks MQPS=88 m, angle, Q, P, S, equals, 88, degrees \qquad m \angle RPS = 5x + 34^\circmRPS=5x+34 m, angle, R, P, S, equals, 5, x, plus, 34, degrees \qquad m \angle QPR = 8x + 15^\circmQPR=8x+15 Can someone help please please Biotic components of an ecosystem are alive.-True-False There is a link between the amount you sweat and the amount of urine you produce. How does the body coordinate these two responses? can u please help me 1. What was the Eastern bloc?the part of Europe that was a temporary Soviet occupation zonethe part of Germany and Poland occupied by the Sovietsthe European nations that remained sovereign but became Communistthe European nations that were annexed by the Soviet Union How do you say hello in German Read this sentence from "excerpt from President Richard Nixon's News Conference: The President's Personal Finances." I don't want to tilt either way at the moment, as you can be suresince the question was raised a moment ago about my tax payments. Knowing that the words tilt and level are antonyms, what is the definition of tilt? moderate lean surrender shake During the 1990s, the United States was involved with the politics of the island country of Haiti. How did the United States affect this island's political structure? HURRY PLEASE HELP Caitlin wants to collect toys to donate to a local shelter. Her goal is to collect at least 300 toys. On Monday she collects 40 toys. On Tuesday she collects twice as many as on Monday. On Wednesday she collects three times as many as Monday. Which inequality represents the total number of toys Caitlin must collect on Thursday, x, to reach or exceed her goal of 300 toys? Someone help me find 2161965 please.